Prove that $sumlimits_{k=1}^nlfloor n/krfloor+lfloor sqrt{n} rfloor$ is even. [duplicate]












2












$begingroup$



This question already has an answer here:




  • How to show that $lfloor n/1rfloor+lfloor n/2 rfloor+…+lfloor n/nrfloor+lfloor{sqrt{n}}rfloor$ is even?

    1 answer





Let $n$ be a natural number. Prove that
$displaystyle sum_{k=1}^nlfloor n/krfloor+lfloor sqrt{n} rfloor$ is even.




I tried to introduce the fractional part, but it didnt help me.
Next, I considered an inequality to create a bound, but that too was in vain. Next I found out that $lfloor x rfloor = lfloor x/2 rfloor +lfloor x+1/2 rfloor$.
I applied this, but as all were as a sum, it became hard for me to cancel out and it made my work difficult. Now the next problem is about the square root in the box. I once saw in an example that



$$lfloor sqrt{n}+ sqrt{n+1} rfloor = sqrt{4n+1}$$



Now even if this may seem useful, I cannot understand how to remove the $lfloor sqrt{n+1} rfloor $ from the identity. Any help would be helpful!










share|cite|improve this question











$endgroup$



marked as duplicate by rtybase, Community Jan 21 at 15:50


This question has been asked before and already has an answer. If those answers do not fully address your question, please ask a new question.














  • 2




    $begingroup$
    Please format your question through latex.
    $endgroup$
    – lightxbulb
    Jan 21 at 15:17










  • $begingroup$
    I do not know how to use latexx
    $endgroup$
    – user636268
    Jan 21 at 15:20






  • 1




    $begingroup$
    math.meta.stackexchange.com/questions/5020/…
    $endgroup$
    – lightxbulb
    Jan 21 at 15:20










  • $begingroup$
    Do you mean $lfloor x rfloor$? You did mention largest integer not exceeding $x$.
    $endgroup$
    – Mohammad Zuhair Khan
    Jan 21 at 15:21












  • $begingroup$
    Yes the floor function
    $endgroup$
    – user636268
    Jan 21 at 15:22
















2












$begingroup$



This question already has an answer here:




  • How to show that $lfloor n/1rfloor+lfloor n/2 rfloor+…+lfloor n/nrfloor+lfloor{sqrt{n}}rfloor$ is even?

    1 answer





Let $n$ be a natural number. Prove that
$displaystyle sum_{k=1}^nlfloor n/krfloor+lfloor sqrt{n} rfloor$ is even.




I tried to introduce the fractional part, but it didnt help me.
Next, I considered an inequality to create a bound, but that too was in vain. Next I found out that $lfloor x rfloor = lfloor x/2 rfloor +lfloor x+1/2 rfloor$.
I applied this, but as all were as a sum, it became hard for me to cancel out and it made my work difficult. Now the next problem is about the square root in the box. I once saw in an example that



$$lfloor sqrt{n}+ sqrt{n+1} rfloor = sqrt{4n+1}$$



Now even if this may seem useful, I cannot understand how to remove the $lfloor sqrt{n+1} rfloor $ from the identity. Any help would be helpful!










share|cite|improve this question











$endgroup$



marked as duplicate by rtybase, Community Jan 21 at 15:50


This question has been asked before and already has an answer. If those answers do not fully address your question, please ask a new question.














  • 2




    $begingroup$
    Please format your question through latex.
    $endgroup$
    – lightxbulb
    Jan 21 at 15:17










  • $begingroup$
    I do not know how to use latexx
    $endgroup$
    – user636268
    Jan 21 at 15:20






  • 1




    $begingroup$
    math.meta.stackexchange.com/questions/5020/…
    $endgroup$
    – lightxbulb
    Jan 21 at 15:20










  • $begingroup$
    Do you mean $lfloor x rfloor$? You did mention largest integer not exceeding $x$.
    $endgroup$
    – Mohammad Zuhair Khan
    Jan 21 at 15:21












  • $begingroup$
    Yes the floor function
    $endgroup$
    – user636268
    Jan 21 at 15:22














2












2








2





$begingroup$



This question already has an answer here:




  • How to show that $lfloor n/1rfloor+lfloor n/2 rfloor+…+lfloor n/nrfloor+lfloor{sqrt{n}}rfloor$ is even?

    1 answer





Let $n$ be a natural number. Prove that
$displaystyle sum_{k=1}^nlfloor n/krfloor+lfloor sqrt{n} rfloor$ is even.




I tried to introduce the fractional part, but it didnt help me.
Next, I considered an inequality to create a bound, but that too was in vain. Next I found out that $lfloor x rfloor = lfloor x/2 rfloor +lfloor x+1/2 rfloor$.
I applied this, but as all were as a sum, it became hard for me to cancel out and it made my work difficult. Now the next problem is about the square root in the box. I once saw in an example that



$$lfloor sqrt{n}+ sqrt{n+1} rfloor = sqrt{4n+1}$$



Now even if this may seem useful, I cannot understand how to remove the $lfloor sqrt{n+1} rfloor $ from the identity. Any help would be helpful!










share|cite|improve this question











$endgroup$





This question already has an answer here:




  • How to show that $lfloor n/1rfloor+lfloor n/2 rfloor+…+lfloor n/nrfloor+lfloor{sqrt{n}}rfloor$ is even?

    1 answer





Let $n$ be a natural number. Prove that
$displaystyle sum_{k=1}^nlfloor n/krfloor+lfloor sqrt{n} rfloor$ is even.




I tried to introduce the fractional part, but it didnt help me.
Next, I considered an inequality to create a bound, but that too was in vain. Next I found out that $lfloor x rfloor = lfloor x/2 rfloor +lfloor x+1/2 rfloor$.
I applied this, but as all were as a sum, it became hard for me to cancel out and it made my work difficult. Now the next problem is about the square root in the box. I once saw in an example that



$$lfloor sqrt{n}+ sqrt{n+1} rfloor = sqrt{4n+1}$$



Now even if this may seem useful, I cannot understand how to remove the $lfloor sqrt{n+1} rfloor $ from the identity. Any help would be helpful!





This question already has an answer here:




  • How to show that $lfloor n/1rfloor+lfloor n/2 rfloor+…+lfloor n/nrfloor+lfloor{sqrt{n}}rfloor$ is even?

    1 answer








elementary-number-theory summation floor-function






share|cite|improve this question















share|cite|improve this question













share|cite|improve this question




share|cite|improve this question








edited Feb 1 at 13:46









Martin Sleziak

44.8k10119273




44.8k10119273










asked Jan 21 at 15:14







user636268











marked as duplicate by rtybase, Community Jan 21 at 15:50


This question has been asked before and already has an answer. If those answers do not fully address your question, please ask a new question.









marked as duplicate by rtybase, Community Jan 21 at 15:50


This question has been asked before and already has an answer. If those answers do not fully address your question, please ask a new question.










  • 2




    $begingroup$
    Please format your question through latex.
    $endgroup$
    – lightxbulb
    Jan 21 at 15:17










  • $begingroup$
    I do not know how to use latexx
    $endgroup$
    – user636268
    Jan 21 at 15:20






  • 1




    $begingroup$
    math.meta.stackexchange.com/questions/5020/…
    $endgroup$
    – lightxbulb
    Jan 21 at 15:20










  • $begingroup$
    Do you mean $lfloor x rfloor$? You did mention largest integer not exceeding $x$.
    $endgroup$
    – Mohammad Zuhair Khan
    Jan 21 at 15:21












  • $begingroup$
    Yes the floor function
    $endgroup$
    – user636268
    Jan 21 at 15:22














  • 2




    $begingroup$
    Please format your question through latex.
    $endgroup$
    – lightxbulb
    Jan 21 at 15:17










  • $begingroup$
    I do not know how to use latexx
    $endgroup$
    – user636268
    Jan 21 at 15:20






  • 1




    $begingroup$
    math.meta.stackexchange.com/questions/5020/…
    $endgroup$
    – lightxbulb
    Jan 21 at 15:20










  • $begingroup$
    Do you mean $lfloor x rfloor$? You did mention largest integer not exceeding $x$.
    $endgroup$
    – Mohammad Zuhair Khan
    Jan 21 at 15:21












  • $begingroup$
    Yes the floor function
    $endgroup$
    – user636268
    Jan 21 at 15:22








2




2




$begingroup$
Please format your question through latex.
$endgroup$
– lightxbulb
Jan 21 at 15:17




$begingroup$
Please format your question through latex.
$endgroup$
– lightxbulb
Jan 21 at 15:17












$begingroup$
I do not know how to use latexx
$endgroup$
– user636268
Jan 21 at 15:20




$begingroup$
I do not know how to use latexx
$endgroup$
– user636268
Jan 21 at 15:20




1




1




$begingroup$
math.meta.stackexchange.com/questions/5020/…
$endgroup$
– lightxbulb
Jan 21 at 15:20




$begingroup$
math.meta.stackexchange.com/questions/5020/…
$endgroup$
– lightxbulb
Jan 21 at 15:20












$begingroup$
Do you mean $lfloor x rfloor$? You did mention largest integer not exceeding $x$.
$endgroup$
– Mohammad Zuhair Khan
Jan 21 at 15:21






$begingroup$
Do you mean $lfloor x rfloor$? You did mention largest integer not exceeding $x$.
$endgroup$
– Mohammad Zuhair Khan
Jan 21 at 15:21














$begingroup$
Yes the floor function
$endgroup$
– user636268
Jan 21 at 15:22




$begingroup$
Yes the floor function
$endgroup$
– user636268
Jan 21 at 15:22










2 Answers
2






active

oldest

votes


















0












$begingroup$

Since $lfloor sqrt{n}rfloor^2$ and $lfloor sqrt{n}rfloor$ have the same parity it suffices to show the following identity
$$sum_{k=1}^nlfloor n/krfloor=2sum_{k=1}^{lfloor sqrt{n}rfloor} lfloor n/krfloor-lfloor sqrt{n}rfloor^2.
$$

See Hurkyl's answer to Simplifying $sum_{i=1}^n{lfloor frac{n}{i} rfloor}$?



In other words, in a direct way,
$$begin{align}
sum_{k=1}^n lfloor n/k rfloor
&= sum_{j,k} [1 leq j] [1 leq k] [jk leq n]\
&=underbrace{sum_{jnot=k} [1 leq j] [1 leq k] [jk leq n]}_{text{even}}+sum_{j=k} [1 leq j] [1 leq k] [jk leq n]\
&equiv sum_{j=k} [1 leq j] [1 leq k] [jk leq n] =lfloor sqrt{n}rfloorpmod{2}end{align}$$

where $[P]$ is $1$ if $P$ is true, and $0$ if false.






share|cite|improve this answer











$endgroup$













  • $begingroup$
    How do you get that?
    $endgroup$
    – user636268
    Jan 21 at 15:34






  • 1




    $begingroup$
    This is my hint... ;-)
    $endgroup$
    – Robert Z
    Jan 21 at 15:36






  • 1




    $begingroup$
    You knew this identity from the beginning?😲
    $endgroup$
    – user636268
    Jan 21 at 15:37






  • 1




    $begingroup$
    @Md.ShamimAkhtar See my edit.
    $endgroup$
    – Robert Z
    Jan 21 at 15:44






  • 1




    $begingroup$
    @Md.ShamimAkhtar I gave you a direct way.
    $endgroup$
    – Robert Z
    Jan 21 at 16:06



















1












$begingroup$

Hint: try to prove this by induction on $n$. When you replace $n$ by $n+1$, some terms in the expression will increase by $1$, count how many do for each $n$.






share|cite|improve this answer









$endgroup$



















    2 Answers
    2






    active

    oldest

    votes








    2 Answers
    2






    active

    oldest

    votes









    active

    oldest

    votes






    active

    oldest

    votes









    0












    $begingroup$

    Since $lfloor sqrt{n}rfloor^2$ and $lfloor sqrt{n}rfloor$ have the same parity it suffices to show the following identity
    $$sum_{k=1}^nlfloor n/krfloor=2sum_{k=1}^{lfloor sqrt{n}rfloor} lfloor n/krfloor-lfloor sqrt{n}rfloor^2.
    $$

    See Hurkyl's answer to Simplifying $sum_{i=1}^n{lfloor frac{n}{i} rfloor}$?



    In other words, in a direct way,
    $$begin{align}
    sum_{k=1}^n lfloor n/k rfloor
    &= sum_{j,k} [1 leq j] [1 leq k] [jk leq n]\
    &=underbrace{sum_{jnot=k} [1 leq j] [1 leq k] [jk leq n]}_{text{even}}+sum_{j=k} [1 leq j] [1 leq k] [jk leq n]\
    &equiv sum_{j=k} [1 leq j] [1 leq k] [jk leq n] =lfloor sqrt{n}rfloorpmod{2}end{align}$$

    where $[P]$ is $1$ if $P$ is true, and $0$ if false.






    share|cite|improve this answer











    $endgroup$













    • $begingroup$
      How do you get that?
      $endgroup$
      – user636268
      Jan 21 at 15:34






    • 1




      $begingroup$
      This is my hint... ;-)
      $endgroup$
      – Robert Z
      Jan 21 at 15:36






    • 1




      $begingroup$
      You knew this identity from the beginning?😲
      $endgroup$
      – user636268
      Jan 21 at 15:37






    • 1




      $begingroup$
      @Md.ShamimAkhtar See my edit.
      $endgroup$
      – Robert Z
      Jan 21 at 15:44






    • 1




      $begingroup$
      @Md.ShamimAkhtar I gave you a direct way.
      $endgroup$
      – Robert Z
      Jan 21 at 16:06
















    0












    $begingroup$

    Since $lfloor sqrt{n}rfloor^2$ and $lfloor sqrt{n}rfloor$ have the same parity it suffices to show the following identity
    $$sum_{k=1}^nlfloor n/krfloor=2sum_{k=1}^{lfloor sqrt{n}rfloor} lfloor n/krfloor-lfloor sqrt{n}rfloor^2.
    $$

    See Hurkyl's answer to Simplifying $sum_{i=1}^n{lfloor frac{n}{i} rfloor}$?



    In other words, in a direct way,
    $$begin{align}
    sum_{k=1}^n lfloor n/k rfloor
    &= sum_{j,k} [1 leq j] [1 leq k] [jk leq n]\
    &=underbrace{sum_{jnot=k} [1 leq j] [1 leq k] [jk leq n]}_{text{even}}+sum_{j=k} [1 leq j] [1 leq k] [jk leq n]\
    &equiv sum_{j=k} [1 leq j] [1 leq k] [jk leq n] =lfloor sqrt{n}rfloorpmod{2}end{align}$$

    where $[P]$ is $1$ if $P$ is true, and $0$ if false.






    share|cite|improve this answer











    $endgroup$













    • $begingroup$
      How do you get that?
      $endgroup$
      – user636268
      Jan 21 at 15:34






    • 1




      $begingroup$
      This is my hint... ;-)
      $endgroup$
      – Robert Z
      Jan 21 at 15:36






    • 1




      $begingroup$
      You knew this identity from the beginning?😲
      $endgroup$
      – user636268
      Jan 21 at 15:37






    • 1




      $begingroup$
      @Md.ShamimAkhtar See my edit.
      $endgroup$
      – Robert Z
      Jan 21 at 15:44






    • 1




      $begingroup$
      @Md.ShamimAkhtar I gave you a direct way.
      $endgroup$
      – Robert Z
      Jan 21 at 16:06














    0












    0








    0





    $begingroup$

    Since $lfloor sqrt{n}rfloor^2$ and $lfloor sqrt{n}rfloor$ have the same parity it suffices to show the following identity
    $$sum_{k=1}^nlfloor n/krfloor=2sum_{k=1}^{lfloor sqrt{n}rfloor} lfloor n/krfloor-lfloor sqrt{n}rfloor^2.
    $$

    See Hurkyl's answer to Simplifying $sum_{i=1}^n{lfloor frac{n}{i} rfloor}$?



    In other words, in a direct way,
    $$begin{align}
    sum_{k=1}^n lfloor n/k rfloor
    &= sum_{j,k} [1 leq j] [1 leq k] [jk leq n]\
    &=underbrace{sum_{jnot=k} [1 leq j] [1 leq k] [jk leq n]}_{text{even}}+sum_{j=k} [1 leq j] [1 leq k] [jk leq n]\
    &equiv sum_{j=k} [1 leq j] [1 leq k] [jk leq n] =lfloor sqrt{n}rfloorpmod{2}end{align}$$

    where $[P]$ is $1$ if $P$ is true, and $0$ if false.






    share|cite|improve this answer











    $endgroup$



    Since $lfloor sqrt{n}rfloor^2$ and $lfloor sqrt{n}rfloor$ have the same parity it suffices to show the following identity
    $$sum_{k=1}^nlfloor n/krfloor=2sum_{k=1}^{lfloor sqrt{n}rfloor} lfloor n/krfloor-lfloor sqrt{n}rfloor^2.
    $$

    See Hurkyl's answer to Simplifying $sum_{i=1}^n{lfloor frac{n}{i} rfloor}$?



    In other words, in a direct way,
    $$begin{align}
    sum_{k=1}^n lfloor n/k rfloor
    &= sum_{j,k} [1 leq j] [1 leq k] [jk leq n]\
    &=underbrace{sum_{jnot=k} [1 leq j] [1 leq k] [jk leq n]}_{text{even}}+sum_{j=k} [1 leq j] [1 leq k] [jk leq n]\
    &equiv sum_{j=k} [1 leq j] [1 leq k] [jk leq n] =lfloor sqrt{n}rfloorpmod{2}end{align}$$

    where $[P]$ is $1$ if $P$ is true, and $0$ if false.







    share|cite|improve this answer














    share|cite|improve this answer



    share|cite|improve this answer








    edited Jan 21 at 16:06

























    answered Jan 21 at 15:31









    Robert ZRobert Z

    99.8k1068140




    99.8k1068140












    • $begingroup$
      How do you get that?
      $endgroup$
      – user636268
      Jan 21 at 15:34






    • 1




      $begingroup$
      This is my hint... ;-)
      $endgroup$
      – Robert Z
      Jan 21 at 15:36






    • 1




      $begingroup$
      You knew this identity from the beginning?😲
      $endgroup$
      – user636268
      Jan 21 at 15:37






    • 1




      $begingroup$
      @Md.ShamimAkhtar See my edit.
      $endgroup$
      – Robert Z
      Jan 21 at 15:44






    • 1




      $begingroup$
      @Md.ShamimAkhtar I gave you a direct way.
      $endgroup$
      – Robert Z
      Jan 21 at 16:06


















    • $begingroup$
      How do you get that?
      $endgroup$
      – user636268
      Jan 21 at 15:34






    • 1




      $begingroup$
      This is my hint... ;-)
      $endgroup$
      – Robert Z
      Jan 21 at 15:36






    • 1




      $begingroup$
      You knew this identity from the beginning?😲
      $endgroup$
      – user636268
      Jan 21 at 15:37






    • 1




      $begingroup$
      @Md.ShamimAkhtar See my edit.
      $endgroup$
      – Robert Z
      Jan 21 at 15:44






    • 1




      $begingroup$
      @Md.ShamimAkhtar I gave you a direct way.
      $endgroup$
      – Robert Z
      Jan 21 at 16:06
















    $begingroup$
    How do you get that?
    $endgroup$
    – user636268
    Jan 21 at 15:34




    $begingroup$
    How do you get that?
    $endgroup$
    – user636268
    Jan 21 at 15:34




    1




    1




    $begingroup$
    This is my hint... ;-)
    $endgroup$
    – Robert Z
    Jan 21 at 15:36




    $begingroup$
    This is my hint... ;-)
    $endgroup$
    – Robert Z
    Jan 21 at 15:36




    1




    1




    $begingroup$
    You knew this identity from the beginning?😲
    $endgroup$
    – user636268
    Jan 21 at 15:37




    $begingroup$
    You knew this identity from the beginning?😲
    $endgroup$
    – user636268
    Jan 21 at 15:37




    1




    1




    $begingroup$
    @Md.ShamimAkhtar See my edit.
    $endgroup$
    – Robert Z
    Jan 21 at 15:44




    $begingroup$
    @Md.ShamimAkhtar See my edit.
    $endgroup$
    – Robert Z
    Jan 21 at 15:44




    1




    1




    $begingroup$
    @Md.ShamimAkhtar I gave you a direct way.
    $endgroup$
    – Robert Z
    Jan 21 at 16:06




    $begingroup$
    @Md.ShamimAkhtar I gave you a direct way.
    $endgroup$
    – Robert Z
    Jan 21 at 16:06











    1












    $begingroup$

    Hint: try to prove this by induction on $n$. When you replace $n$ by $n+1$, some terms in the expression will increase by $1$, count how many do for each $n$.






    share|cite|improve this answer









    $endgroup$


















      1












      $begingroup$

      Hint: try to prove this by induction on $n$. When you replace $n$ by $n+1$, some terms in the expression will increase by $1$, count how many do for each $n$.






      share|cite|improve this answer









      $endgroup$
















        1












        1








        1





        $begingroup$

        Hint: try to prove this by induction on $n$. When you replace $n$ by $n+1$, some terms in the expression will increase by $1$, count how many do for each $n$.






        share|cite|improve this answer









        $endgroup$



        Hint: try to prove this by induction on $n$. When you replace $n$ by $n+1$, some terms in the expression will increase by $1$, count how many do for each $n$.







        share|cite|improve this answer












        share|cite|improve this answer



        share|cite|improve this answer










        answered Jan 21 at 15:38









        WojowuWojowu

        18.8k23172




        18.8k23172















            Popular posts from this blog

            Can a sorcerer learn a 5th-level spell early by creating spell slots using the Font of Magic feature?

            ts Property 'filter' does not exist on type '{}'

            mat-slide-toggle shouldn't change it's state when I click cancel in confirmation window